www.vorkurse.de
Ein Projekt von vorhilfe.de
Die Online-Kurse der Vorhilfe

E-Learning leicht gemacht.
Hallo Gast!einloggen | registrieren ]
Startseite · Mitglieder · Teams · Forum · Wissen · Kurse · Impressum
Forenbaum
^ Forenbaum
Status Mathe-Vorkurse
  Status Organisatorisches
  Status Schule
    Status Wiederholung Algebra
    Status Einführung Analysis
    Status Einführung Analytisc
    Status VK 21: Mathematik 6.
    Status VK 37: Kurvendiskussionen
    Status VK Abivorbereitungen
  Status Universität
    Status Lerngruppe LinAlg
    Status VK 13 Analysis I FH
    Status Algebra 2006
    Status VK 22: Algebra 2007
    Status GruMiHH 06
    Status VK 58: Algebra 1
    Status VK 59: Lineare Algebra
    Status VK 60: Analysis
    Status Wahrscheinlichkeitst

Gezeigt werden alle Foren bis zur Tiefe 2

Navigation
 Startseite...
 Neuerdings beta neu
 Forum...
 vorwissen...
 vorkurse...
 Werkzeuge...
 Nachhilfevermittlung beta...
 Online-Spiele beta
 Suchen
 Verein...
 Impressum
Das Projekt
Server und Internetanbindung werden durch Spenden finanziert.
Organisiert wird das Projekt von unserem Koordinatorenteam.
Hunderte Mitglieder helfen ehrenamtlich in unseren moderierten Foren.
Anbieter der Seite ist der gemeinnützige Verein "Vorhilfe.de e.V.".
Partnerseiten
Weitere Fächer:

Open Source FunktionenplotterFunkyPlot: Kostenloser und quelloffener Funktionenplotter für Linux und andere Betriebssysteme
Forum "Trigonometrische Funktionen" - Polarkoordinatentransformation
Polarkoordinatentransformation < Trigonometr. Fktn < Analysis < Oberstufe < Schule < Mathe < Vorhilfe
Ansicht: [ geschachtelt ] | ^ Forum "Trigonometrische Funktionen"  | ^^ Alle Foren  | ^ Forenbaum  | Materialien

Polarkoordinatentransformation: Frage (beantwortet)
Status: (Frage) beantwortet Status 
Datum: 17:14 Do 01.05.2008
Autor: tedd

Aufgabe
Transformieren sie gegeben Funktion in Polarkoordinaten.

Hi!
Ich habe Probleme folgende Funktion in Polarkoordinaten zu transformieren:
[mm](x^2+y^2)^2-4*x^3+12*x*y^2=0[/mm]

also mit
[mm]r=\sqrt{x^2+y^2}[/mm]
[mm]r^2=x^2+y^2[/mm]
[mm]x=\cos\phi*r[/mm] und
[mm]y=\sin\phi*r[/mm]

komme ich auf
[mm]r^4-4*(\cos\phi*r)^3+12*\cos\phi*r*(\sin\phi*r)^2=0[/mm]
aber von da komme ich irgendwie nicht weiter.
Als Ergebnis soll wohl
[mm]r=4*\cos(3*\phi)[/mm] rauskommen.
Wie rechne ich jetzt weiter oder was muss ich anders machen?
Danke schonmal im vorraus und beste Grüße,
tedd ;)

        
Bezug
Polarkoordinatentransformation: 2 Tipps
Status: (Antwort) fertig Status 
Datum: 17:18 Do 01.05.2008
Autor: Loddar

Hallo tedd!


Verwende hier [mm] $\sin^2(\phi) [/mm] \ = \ [mm] 1-\cos^2(\phi)$ [/mm] sowie das Additionstheorem [mm] $\cos(3*\phi) [/mm] \ = \ [mm] 4*\cos^3(\phi)-3*\cos(\phi)$ [/mm] .


Gruß
Loddar


Bezug
                
Bezug
Polarkoordinatentransformation: Frage (beantwortet)
Status: (Frage) beantwortet Status 
Datum: 21:50 Do 01.05.2008
Autor: tedd

Hey Loddar, danke für die Antwort.
Die beiden Sachen helfen mir weiter. Der trigonometrische Pythagoras ist mir klar aber wie ich mit den Additionstheoremen auf [mm]\cos(3*\phi) = 4*\cos^3(\phi)-3*\cos(\phi)[/mm] ist mir nicht ganz klar. Wäre sehr dankbar wenn man mir das nochmal erklären könnte :)
Danke  und Gruß,
tedd

Bezug
                        
Bezug
Polarkoordinatentransformation: Antwort
Status: (Antwort) fertig Status 
Datum: 22:13 Do 01.05.2008
Autor: schachuzipus

Eingabefehler: "\left" und "\right" müssen immer paarweise auftreten, es wurde aber ein Teil ohne Entsprechung gefunden (siehe rote Markierung)

Hallo tedd,

verwende 2mal die Additionstheoreme für $\sin$ und $\cos$ und die Beziehung $\sin^2(x)+\cos^2(x)=1$


$\cos(3\phi)=\cos(2\phi+\phi)=\red{\cos(2\phi)}\cos(\phi)-\blue{\sin(2\phi)}\sin(\phi) \qquad $ Additionstheorem für $\cos$

$=\red{[\cos(\phi)\cos(\phi)-\sin(\phi)\sin(\phi)]}\cos(\phi)-\blue{\left[\\sin(\phi)\cos(\phi)+\sin(\phi)\cos(\phi)]}\sin(\phi) \qquad$ Additionstheorem für $\cos$ und  $\sin$

$=[\cos^2(\phi)-\green{\sin^2(\phi)}]\cos(\phi)-2\cos(\phi)\green{\sin^2(\phi)}$

$=[\cos^2(\phi)-\green{(1-\cos^2(\phi))}]\cos(\phi)-2\cos(\phi)\green{(1-\cos^2(\phi))}$

Das fasse nun mal alles nett zusammen....


Gruß

schachuzipus



Bezug
                                
Bezug
Polarkoordinatentransformation: Frage (beantwortet)
Status: (Frage) beantwortet Status 
Datum: 12:38 Sa 03.05.2008
Autor: tedd

Danke!
Jetzt kann ichs nachvollziehen :)
Habs so zusammengefasst:
[mm]cos(3\phi)[/mm]
[mm]=[\cos^2(\phi)-(1-\cos(\phi))]*\cos(\phi)-2*\cos(\phi) (1-\cos^2(\phi))[/mm]
[mm]=(2*\cos^2(\phi)-1)*\cos(\phi)-2*\cos(\phi)+2*\cos^3(\phi)[/mm]
[mm]=2*\cos^3(\phi)-\cos(\phi)-2*\cos(\phi)+2*\cos^3(\phi)[/mm]
[mm]=4*\cos^3(\phi)-3*\cos(\phi)[/mm]

Hab die gesamte Aufgabe dann so gelöst:

[mm] (x^2+y^2)^2-4*x^3+12*xy^2=0 [/mm]
[mm] r^4-4*\cos^3(\phi)*r^3+12*\cos(\phi)*r*sin^2(\phi)*r^2=0 [/mm]
[mm] r^3*(r-4*\cos^3(\phi)+12*\cos(\phi)*\sin^2(\phi))=0 [/mm]
[mm] r^3*(r-4*\cos^3(\phi)+12*\cos(\phi)*(1-\cos^2(\phi))=0 [/mm]
[mm] r^3*(r-4*\cos^3(\phi)+12*\cos(\phi)-12*\cos^3(\phi))=0 [/mm]
[mm] -r^3*(-r+4*\cos^3(\phi)-3*\cos(\phi)+4*\cos^3(\phi)-3*\cos(\phi)+4*\cos^3(\phi)-3*\cos(\phi)+4*\cos^3(\phi)-3*\cos(\phi))=0 [/mm]
[mm] -r^3*(-r+4*\cos(3\phi))=0 [/mm]
[mm] r^4-4*\cos(3\phi)*r^3=0 [/mm]
[mm] r^4=4*\cos(3\phi)*r^3 [/mm]
[mm] r=4*\cos(3\phi) [/mm]

Ich nehm an es wird so rihtig sein. Danke für eure Hilfe ihr 2.
Beste Grüße,
tedd


Ahh verzeihung ich habe den Artikel versehentlich als weitere Frage gepostet

Bezug
                                        
Bezug
Polarkoordinatentransformation: Antwort
Status: (Antwort) fertig Status 
Datum: 12:52 Sa 03.05.2008
Autor: MathePower

Hallo tedd,

> Danke!
>  Jetzt kann ichs nachvollziehen :)
>  Habs so zusammengefasst:
>  [mm]cos(3\phi)[/mm]
>  [mm]=[\cos^2(\phi)-(1-\cos(\phi))]*\cos(\phi)-2*\cos(\phi) (1-\cos^2(\phi))[/mm]
>  
> [mm]=(2*\cos^2(\phi)-1)*\cos(\phi)-2*\cos(\phi)+2*\cos^3(\phi)[/mm]
>  [mm]=2*\cos^3(\phi)-\cos(\phi)-2*\cos(\phi)+2*\cos^3(\phi)[/mm]
>  [mm]=4*\cos^3(\phi)-3*\cos(\phi)[/mm]
>  
> Hab die gesamte Aufgabe dann so gelöst:
>  
> [mm](x^2+y^2)^2-4*x^3+12*xy^2=0[/mm]
>  [mm]r^4-4*\cos^3(\phi)*r^3+12*\cos(\phi)*r*sin^2(\phi)*r^2=0[/mm]
>  [mm]r^3*(r-4*\cos^3(\phi)+12*\cos(\phi)*\sin^2(\phi))=0[/mm]
>  [mm]r^3*(r-4*\cos^3(\phi)+12*\cos(\phi)*(1-\cos^2(\phi))=0[/mm]
>  [mm]r^3*(r-4*\cos^3(\phi)+12*\cos(\phi)-12*\cos^3(\phi))=0[/mm]
>  
> [mm]-r^3*(-r+4*\cos^3(\phi)-3*\cos(\phi)+4*\cos^3(\phi)-3*\cos(\phi)+4*\cos^3(\phi)-3*\cos(\phi)+4*\cos^3(\phi)-3*\cos(\phi))=0[/mm]
>  [mm]-r^3*(-r+4*\cos(3\phi))=0[/mm]
>  [mm]r^4-4*\cos(3\phi)*r^3=0[/mm]
>  [mm]r^4=4*\cos(3\phi)*r^3[/mm]
>  [mm]r=4*\cos(3\phi)[/mm]


Stimmt. [ok]

>  
> Ich nehm an es wird so rihtig sein. Danke für eure Hilfe
> ihr 2.
>  Beste Grüße,
>  tedd
>  
> Ahh verzeihung ich habe den Artikel versehentlich als
> weitere Frage gepostet

Gruß
MathePower

Bezug
Ansicht: [ geschachtelt ] | ^ Forum "Trigonometrische Funktionen"  | ^^ Alle Foren  | ^ Forenbaum  | Materialien


^ Seitenanfang ^
www.vorkurse.de
[ Startseite | Mitglieder | Teams | Forum | Wissen | Kurse | Impressum ]